Probleme bei Aufgaben mit logischen Denkenn?

1bsp. - (Schule, Mathematik, Leben) 2bsp. - (Schule, Mathematik, Leben)

6 Antworten

Links übersetzung

Das große rad dreht das kleine rad

Der umfang des großen rades ist

U=2*pi*R_groß

Der umfang des kleinen ist

U=2*pi*R_klein

Jetzt kannst du dir mal die umfänge der räder als strecken vorstellen. Wenn sich das große rad einmal dreht muss sich das kleinere rad mehrmals drehen.


KingMayweather 
Beitragsersteller
 29.01.2019, 16:53

Bei mehreren dreht sich also mit hoher Wahrscheinlichkeit das kleinst am schnellsten? Ps: Danke schonmal rettest mich

1

Inc... Antwort zur zweiten Aufgabe.

Das Kleinste, "b" . Ist wie beim Fahrad... Vorne langsam...(a) höchster Gang, aber hinten schnell.

Bretter... Die Aufgabe ist unklar. Entscheidend ist, von wo die Kraft kommt. Ausgegangen davon, das die Kraft von von oben nach unten wirkt, dann das wo das schräge Stützbrett in der Mitte vom Waagerechten. angebracht wird.

Also A oder B... Die Zeichnungen sind nämlich beide gleich.

Es fehlt ein Bild, bei dem das Stützbrett näher an der schwarzen ... ich denke mal... Wand, montiert ist.

Die wichtigste Angabe fehlt ohnehin... Wo liegt der Gegenstand überhaupt auf dem Regal? Nahe an der Wand? Dann wäre C sogar richtig. Die Mitte ist nur richtig wenn das Gewicht gleichmäßig über das Regal vrteilt ist und der Schwerpunkt in der Mitte liegt.

Wie kann man nur so eine unklare Aufgabe stellen?

Dein Lehrer sollte sich mal seine Aufgaben selber ansehen, bevor er so einen Mist verteilt.

Zeige meine Antwort mal Deinem Lehrer!

Zeige Deinem Lehrer das mal.

Mario


oetschai  29.01.2019, 19:21

Räder: beim Rad "b" sind ja wohl die beiden Scheiben starr miteinander verbunden - ansonsten könnte man überhaupt keine eindeutige Lösung angeben

Regale: WO sich in den Beispielen die Last befindet, spielt für die Lösung keine Rolle! Die Frage ist lediglich, welches System der Kräfteverteilung das höchste Gewicht tragen kann. Und da ist die Variante "a" eindeutig zu präferieren (und ja, ich habe schon genug Regale an diverseste Wände gedübelt, um diesbezüglich eine gewisse Kompetenz zugesprochen zu bekommen...)

2
techniker68  29.01.2019, 20:39
@oetschai

Oetschai,

Wenn man man den Stütztäger zu kurz baut und ganz außen eine Kraft ansetzt, wird jeh nach Kraft das Brett einfach abbrechen.

Um das zu verstehen muss man nicht studiert haben wie ich.

Da langt eigentlich Vorstellungskraft. Gesunder Menschenverstand.

Der Schwerpunkt der Auflagekraft ist das Ein und alles.

Mario

0
oetschai  29.01.2019, 22:51
@techniker68

Sorry, aber du kämpfst hier gegen Windmühlen, die nicht einmal vorhanden sind...

Dass die größte Belastung für das Regalsystem (Trageseil oder -draht von oben - Regalbrett - Stützelement von unten) dann auftritt, wenn der Schwerpunkt der Last sich möglichst weit weg von der Wand befindet, ist nicht Gegenstand der Aufgabe.

Auch nicht die "richtige" Lastverteilung auf dem Brett oder ein zu kurzes Stützelement...

Es geht allein darum, anzugeben, welches der 3 "angebotenen" und schematisch dargestellten Tragesysteme a, b oder c generell am tragfähigsten ist - mehr nicht.

0
SandyDM  29.01.2019, 18:08

A und B sind nicht gleich. Da sind zwar die stützen im gleichen Winkel angebracht aber nicht die Aufhängungen. Da ist bei A der Winkel größer. Somit sollte A am stabilsten sein.

Und bei der Aufgabe mit den Rädern würde ich schmarotzer2014 recht geben. C

0
techniker68  29.01.2019, 19:41
@SandyDM

Ohne Auflagepunkt der Kraft macht aufgabe 2 Keinen Sinn.

Statik .. ich kenne mich damit aus.

Mario

0

Bei der balken aufgabe musst du ne kräfte billianz machen. Schau dir da mal etwas mechanik an. Dann sollte das kein problem sein. Ich denke die richtige antwort wäre A. Da A die meisten Vertikalkräfte aufnehmen kann. Also auch die gewichtskraft die drauf steht

Dazu musst du den balken zerlegen in F_y und F_x

Dann machste

Summe aller kräfte in X richtung (sollte 0 sein) und Summe aller kräfte in y richtung

Aufgabe 3: Wenn Rad a sich einmal dreht, dreht sich Rad b mehrmals, also ist b schneller als a! Da b ein äußres Rad hat, das größer ist als c, gilt hier dasselbe.

bAußen dreht sich einmal und c dadurch mehr als einmal. Damit ist c am schnellsten.

Naja bei den Rädern denk an ne gangchaltung am Fahrrad. In nem niedrigen gang strampelst Du dir einen ab, fährst aber langsam. Also dreht sich das kleine Rad am schnellsten, da es mehr Umdrehungen brauch um die selbe Strecke zurückzulegen wie ein großes.

Bei den Regalen...kA kenn die physikalischen Gesetzmäßigkeiten nicht genau, bstimmt irgendwas mit helebwirkung oder so, je Größer der Winkel zwischen Brett und Seil desto mehr kann es tragen...mehr als is irgendwie logisch kann ich dazu auch nich sagen xD